Proving the moment of inerta of a sphere: can someone help me please?

  • Thread starter Thread starter vInCEE-j
  • Start date Start date
  • Tags Tags
    Moment Sphere
Click For Summary
The discussion focuses on proving the moment of inertia of a sphere using the formula I = (3m/r^3) * the integral (from 0 to r) of (x^3)((r^2) - (x^2))^(1/2) dx. The original poster is struggling with this proof and seeks assistance. A suggestion is made to use spherical coordinates, specifically x = ρ cos(θ) sin(φ) and dV = ρ^2 sin(φ) dρ dθ dφ, to simplify the calculations. The revised integral involves evaluating ρ^4 cos^2(θ) sin^3(φ) over the specified limits. Overall, the thread emphasizes the importance of coordinate transformation in solving the problem.
vInCEE-j
Messages
3
Reaction score
0
I'm having a difficulty in proving the moment of inertia of a sphere.
Using I=p(x^2)dv, we have to show that

I = (3m/r^3) * the integral (from 0 to r) of (x^3)((r^2) - (x^2))^(1/2) dx

where r=radius, x=the axis

I've been trying to prove it and yet no success. if anyone can show me how to do it, i'd greatly appreciate that.
thanks a lot
 
Physics news on Phys.org
It would seem more reasonable to me to use spherical coordinates:
x= \rhocos(\theta)sin(\phi) and dV= \rho^2 sin(\phi)d\rho\d\thetad\phi
\int\rho x^2dV= p\int_{\phi=0}^\pi\int_{\theta= 0}^{2\pi}\int_{\rho= 0}^R(\rho^3cos^2(\theta)sin^3(\phi)d\rhod\thetad\phi

(oops! Thanks, Fermat.)
 
Last edited by a moderator:
HallsofIvy said:
It would seem more reasonable to me to use spherical coordinates:
x= \rho cos(\theta)sin(\phi) and dV= \rho^2 sin(\phi)d\rho d\theta d\phi
\int\rho x^2\ dV= \rho\int_{\phi=0}^\pi\int_{\theta= 0}^{2\pi}\int_{\rho= 0}^R \rho^4cos^2(\theta)sin^3(\phi) d\rho d\theta d\phi

Ah, now I can see what it says :smile:
 
Last edited:
Question: A clock's minute hand has length 4 and its hour hand has length 3. What is the distance between the tips at the moment when it is increasing most rapidly?(Putnam Exam Question) Answer: Making assumption that both the hands moves at constant angular velocities, the answer is ## \sqrt{7} .## But don't you think this assumption is somewhat doubtful and wrong?

Similar threads

  • · Replies 2 ·
Replies
2
Views
2K
  • · Replies 1 ·
Replies
1
Views
5K
Replies
14
Views
4K
  • · Replies 3 ·
Replies
3
Views
5K
  • · Replies 1 ·
Replies
1
Views
2K
  • · Replies 21 ·
Replies
21
Views
4K
Replies
7
Views
1K
  • · Replies 22 ·
Replies
22
Views
3K
  • · Replies 17 ·
Replies
17
Views
2K
  • · Replies 2 ·
Replies
2
Views
2K